Add solutions for Serway and Jewett v8's chapter 31.
authorW. Trevor King <wking@tremily.us>
Fri, 1 Jun 2012 04:31:08 +0000 (00:31 -0400)
committerW. Trevor King <wking@tremily.us>
Fri, 1 Jun 2012 04:31:08 +0000 (00:31 -0400)
latex/problems/Serway_and_Jewett_8/problem31.23.tex
latex/problems/Serway_and_Jewett_8/problem31.30.tex
latex/problems/Serway_and_Jewett_8/problem31.31.tex
latex/problems/Serway_and_Jewett_8/problem31.45.tex
latex/problems/Serway_and_Jewett_8/problem31.46.tex

index b2618d2fc72c90cb7fb54b5ee837a2b875a1334f..80c2eec278fe2ac06f8bec8da7f3ca9514810e33 100644 (file)
@@ -39,5 +39,49 @@ Dl.draw();
 \end{problem*}
 
 \begin{solution}
-\end{solution}
+\Part{a}
+Moving the bar to the right increases the magnetic flux directed into
+the page by increasing the area of magnetic field enclosed by the
+loop.  Increasing the flux induces a counter-clockwise current to
+resist the change.
+\begin{align}
+  \Phi_B &= AB = lxB \\
+  |\EMF| &= |-\deriv{t}{\Phi_B}| = lB \deriv{t}{x} = lBv \\
+  0 &= \sum_\text{loop} V_i = \EMF - IR \\
+  I &= \frac{\EMF}{R} = \frac{lBv}{R} \;.
+\end{align}
+The counter-clockwise current is moving up through the sliding bar, so
+the magnetic force on the bar is directed to the left, with a
+magnitude of
+\begin{equation}
+  F_B = IlB\sin(90\dg) = IlB = \frac{l^2B^2v}{R} \;.
+\end{equation}
+
+For the bar to move to the right at a constant speed, the net force in
+the $\ihat$ direction should be zero.  We'll have to apply an external
+$F_\text{app}$ to the right to counter $F_B$.
+\begin{align}
+  0 &= \sum_i F_{i,x} = F_\text{app} - F_B \\
+  F_\text{app} &= F_B = \frac{l^2B^2v}{R}
+    = \frac{(1.20\U{m}\cdot2.50\U{T})^2\cdot2.00\U{m/s}}{6.00\U{\Ohm}}
+    = \ans{3.00\U{N}} \;.
+\end{align}
 
+\Part{b}
+The power delivered to a resister is
+\begin{equation}
+  P_R = IV = I^2R = \p({\frac{lBv}{R}})^2 R = \frac{(lBv)^2}{R}
+    = \frac{(1.20\U{m}\cdot2.50\U{T}\cdot2.00\U{m/s})^2}{6.00\U{\Ohm}}
+    = \ans{6.00\U{W}} \;.
+\end{equation}
+
+If you want to look at the problem in terms of energy conservation,
+the external force is putting energy into the system at a rate of
+\begin{equation}
+  P_\text{in} = \deriv{t}{F_\text{app} x} = F_\text{app}\deriv{t}{x}
+    = F_\text{app}v = \frac{l^2B^2v}{R} \cdot v = \frac{(lBv)^2}{R} \;.
+\end{equation}
+All this energy has to go somewhere, and the only place for it to go
+is into resistor heat, so it's no surprise that we get the same
+formula for $P_\text{in}$ that we got for $P_R$.
+\end{solution}
index b441fdd774811d324ba2e481debcfb6ea8edbe1b..d2fb1362e2b547fffae684300418a23d694735e6 100644 (file)
@@ -1,7 +1,7 @@
 \begin{problem*}{31.30}
 A rectangular coil with resistance $R$ has $N$ turns, each of length
 $l$ and width $w$ as shown in Figure~P31.30.  The coil moves into a
-uniform magnetic fiield $\vect{B}$ with constant velocity $\vect{v}$.
+uniform magnetic field $\vect{B}$ with constant velocity $\vect{v}$.
 What are the magnitude and direction of the total magnetic force on
 the coil \Part{a} as it enters the magnetic field, \Part{b} as it
 moves within the field, and \Part{c} as it leaves the field?
@@ -35,5 +35,40 @@ Dw.draw();
 \end{problem*}
 
 \begin{solution}
-\end{solution}
+\Part{a}
+If we define ``into the page'' as the positive direction, the flux
+through the loop will be increasing as the coil enters the field,
+which will induce a current in the counter-clockwise direction
+opposing the changing flux.  The right side of the coil and portions
+of the top and bottom sides will be in the field regions, and because
+of the current will be subject to magnetic forces directed to the
+left, down, and up respectively.  Because equal portions of the top
+and bottom side will be in the field, there will be no vertical
+component in the net force, which will be directed to the left.
+
+The magnitude of the induced \EMF\ is
+\begin{equation}
+  |\EMF| = |-\deriv{t}{\Phi_B}| = \deriv{t}{AB} = B\deriv{t}{Nxw}
+    = NBw\deriv{t}{x} = NBwv \;.
+\end{equation}
+This leads to a current of
+\begin{equation}
+  I = \frac{\EMF}{R} = \frac{Bwv}{R} \;,
+\end{equation}
+which causes a magnetic force of
+\begin{equation}
+  F_B = NIwB\sin(90\dg) = \ans{\frac{N^2B^2w^2v}{R}} \;.
+\end{equation}
 
+\Part{b}
+While the coil is completely inside the field, the flux remains
+constant, so there is no induced current and \ans{no magnetic force}.
+
+\Part{c}
+As the coil leaves the field, the flux drops back towards zero,
+indicing a clockwise current trying to keep the flux up.  Again, the
+vertical components of the resulting magnetic force cancel out, but
+the upward current in the left side will be subject to a magnetic
+force directed to the left.  The magnitude is the same as
+for \Part{a}.
+\end{solution}
index 5cbcc58aa655756e36ce17126a74e4b7be6c1c29..654f3e56003a8fb7e5fe9eef11f5ee7628c1c84d 100644 (file)
@@ -47,5 +47,89 @@ label("$R_2$", (p2.x, yb), align=S);
 \end{problem*}
 
 \begin{solution}
-\end{solution}
+The magnetic flux through the loop will increase as the area enclosed
+increases, inducing \EMF\ in each rod.
+\begin{align}
+  \EMF_1 &= BLv_1 = 4.00\U{mV} \\
+  \EMF_2 &= BLv_2 = 2.00\U{mV} \;.
+\end{align}
+The direction of the induced \EMF{}s will try to resist the increasing
+flux, so $\EMF_1$ will be directed downward and $\EMF_2$ will be
+directed upward.  The situation is then equivalent to the following
+circuit:
+\begin{center}
+\begin{asy}
+import Circ;
+
+real u = 1cm;
 
+MultiTerminal V1 = source(dir=-90, type=DC, Label("$\EMF_1$", align=W));
+MultiTerminal R1 = resistor(V1.terminal[1], dir=-90, Label("$R_1$", align=W));
+MultiTerminal R3 = resistor(dir=90, "$R_3$", draw=false);
+R3.centerto(V1.terminal[0], R1.terminal[1], offset=u);  R3.draw();
+MultiTerminal V2 = source(R1.terminal[1] + (2*u, 0), dir=90, type=DC,
+    Label("$\EMF_2$", align=E));
+MultiTerminal R2 = resistor(V2.terminal[1], dir=90, Label("$R_2$", align=E));
+wire(V1.terminal[0], R2.terminal[1]);
+wire(R1.terminal[1], V2.terminal[0]);
+pair top = (R3.center.x, V1.terminal[0].y);
+pair bot = (top.x, R1.terminal[1].y);
+wire(top, R3.terminal[0]);
+wire(bot, R3.terminal[1]);
+dot(top);
+dot(bot);
+\end{asy}
+\end{center}
+Solve this circuit using the usual Kirchhoff approach.  Label the
+current through the three branches $I_1$, $I_2$, and $I_3$ each
+pointing up, and you have
+\begin{align}
+  I_1 + I_2 + I_3 &= 0 \\
+  \EMF_1 + I_1 R_1 - I_3 R_3 &= 0 \\
+  \EMF_2 - I_2 R_2 + I_3 R_3 &= 0 \;.
+\end{align}
+This gives three equations with three unknowns.  Solve using your
+favorite method.
+\begin{align}
+ \begin{pmatrix}
+  0 \\
+  \EMF_1 \\
+  \EMF_2
+ \end{pmatrix}
+  &=
+ \begin{pmatrix}
+  1\U{\Ohm} & 1\U{\Ohm} & 1\U{\Ohm} \\
+  -R_1 & 0 & R_3 \\
+  0 & R_2 & -R_3
+ \end{pmatrix}
+ \begin{pmatrix}
+  I_1 \\
+  I_2 \\
+  I_3
+ \end{pmatrix} \\
+ \begin{pmatrix}
+  I_1 \\
+  I_2 \\
+  I_3
+ \end{pmatrix}
+  =&
+ \begin{pmatrix}
+  1\U{\Ohm} & 1\U{\Ohm} & 1\U{\Ohm} \\
+  -R_1 & 0 & R_3 \\
+  0 & R_2 & -R_3
+ \end{pmatrix}^{-1}
+ \begin{pmatrix}
+  0 \\
+  \EMF_1 \\
+  \EMF_2
+ \end{pmatrix}
+  =
+ \begin{pmatrix}
+  -327 \\
+  182 \\
+  145
+ \end{pmatrix} \U{$\mu$A}
+ \;.
+\end{align}
+Therefore, \ans{$I_3=145\U{$\mu$A}$ upwards}.
+\end{solution}
index 5acdf74a3c9d2164ba6caea8161eaa9d25967d12..183e4c2828e40fc906c7d40d0fa95f2b885a32d9 100644 (file)
@@ -55,5 +55,21 @@ draw((xr, R.terminal[1].y) -- R.terminal[1]);
 \end{problem*}
 
 \begin{solution}
+Picking up as the positive flux direction, the induced \EMF\ is
+\begin{equation}
+  \EMF = -\deriv{t}{\Phi_B} = -\deriv{t}{NAB} = -NA\deriv{t}{B}
+    = -NA\frac{-2B}{\Delta t} = \frac{2NAB}{\Delta t} \;.
+\end{equation}
+This \EMF\ drives a current through the resistor
+\begin{align}
+  0 &= \EMF - IR \\
+  I &= \frac{\EMF}{R} = \frac{-2NAB}{R\Delta t} \;.
+\end{align}
+Current is defined as the charge passing through a cross section of
+your circuit per unit time, so the charge entering one end of the
+resistor is
+\begin{equation}
+  \Delta q = \frac{\Delta q}{\Delta t} \Delta t = I \Delta t
+    = \frac{2NAB}{R} = \ans{0.880\U{C}} \;.
+\end{equation}
 \end{solution}
-
index e03c4abc4d01c52da7eedce266f770d225a83d7f..6a633513122c33a87f3f294442a7e2231a0bfd69 100644 (file)
@@ -24,5 +24,22 @@ label("$I$", (r+dr, 0), align=E);
 \end{problem*}
 
 \begin{solution}
-\end{solution}
+\Part{a}
+A clockwise current induces an inward magnetic field inside the loop,
+so the external flux must be increasingly out of the page.  Therefore,
+the magnetic field is \ans{increasing}.
 
+\Part{b}
+The induced \EMF\ must be
+\begin{align}
+  0 &= \EMF - IR \\
+  \EMF &= IR \;,
+\end{align}
+This is related to the changing field via magnetic flux.
+\begin{align}
+  |\EMF| &= |-\deriv{t}{\Phi_B}| = \deriv{t}{AB} = A\deriv{t}{B}
+    = \pi r^2 \deriv{t}{B} \\
+  \deriv{t}{B} &= \frac{\EMF}{\pi r^2} = \frac{IR}{\pi r^2}
+    = \ans{62.2\U{T/s}} \;.
+\end{align}
+\end{solution}